IB Physics: Thin Film Interference

  Рет қаралды 34,379

Chris Doner

Chris Doner

Күн бұрын

Пікірлер: 64
@yan_man23
@yan_man23 7 жыл бұрын
Did you mean 0 or 2 inversions at 18:21? Thanks for the videos by the way.
@donerphysics
@donerphysics 7 жыл бұрын
Thanks...should be 0 or 2 inversions.
@aravsriagarwal4056
@aravsriagarwal4056 3 жыл бұрын
@@donerphysics Thanks for the clarification!
@greengoku10
@greengoku10 7 жыл бұрын
Hey, I just wanted to thank you for these videos. You're a brilliant teacher and make my love for physics even greater. I sincerely hope your viewership increases exponentially, because more people need these videos. So thanks again
@MB-zo2xw
@MB-zo2xw 5 жыл бұрын
you're such a god, you explain things that IB expects you to know but are never mentioned in their books
@ananyabeura7224
@ananyabeura7224 4 жыл бұрын
Thank you so much for this. i don't know why the IB expects you to know this without even explaining or mentioning it in the text books
@donerphysics
@donerphysics 4 жыл бұрын
It is thin (some pun intended.)
@raghavcherukuru4936
@raghavcherukuru4936 7 жыл бұрын
Legend, this helped me a lot.
@noenicolas2815
@noenicolas2815 3 жыл бұрын
Exactly what I needed. Perfect explanation
@donerphysics
@donerphysics 3 жыл бұрын
Glad it helped!
@VALR1able
@VALR1able 4 жыл бұрын
This video was very helpful. Thank you very much.
@donerphysics
@donerphysics 4 жыл бұрын
Glad it was helpful!
@neet5080
@neet5080 Жыл бұрын
Great explanation. I was so amazed.
@donerphysics
@donerphysics 10 ай бұрын
Nice to hear!
@abhishekpillai5207
@abhishekpillai5207 Жыл бұрын
Hi! At the question in 19:59, how come the second ray isn't inverted when coming back out from the methyl iodide film to the glass?
@donerphysics
@donerphysics 10 ай бұрын
There is no phase change because the index of refraction of the second material is larger than that of the final material.
@ediewhittington3255
@ediewhittington3255 7 ай бұрын
Thank you! Really well explained!
@donerphysics
@donerphysics 6 ай бұрын
Glad it was helpful!
@longtranhai2761
@longtranhai2761 5 жыл бұрын
The plus a 0.5 for 1 inversion is not included in the formula. Do we just add it ourselves if there is one inversion and in that case will it not be +lambda/2? Thank you for the videos.
@niya.s.
@niya.s. Жыл бұрын
You probably don't care anymore but no you don't need to add the 1/2 for 1 inversion cus it's already included in the formula. In the case of the destructive interference the 1/2 cancels out and then multiplies by lambda on both sides, which is why the IB formula says 2dn=m*lambda. And for constructive interference, the IB formula subtracts by 1/2 on both sides then multiplies by lambda on both sides which is why 2dn = (m + 1/2)*lambda. And, Mr. Doner said to swap the equations for 0 or 2 inversions which makes sense since it'd be the equations if 1/2 wasn't add to 2dn/lambda.
@ashishsehrawat9091
@ashishsehrawat9091 4 жыл бұрын
You are awesome sir
@donerphysics
@donerphysics 3 жыл бұрын
Thank you. Please spread the word about the channel, become a subscriber or a member.
@spifuntastic621
@spifuntastic621 Жыл бұрын
20:34, didn't you say we have to convert the wavelength value according to the medium? So shouldn't we convert the wavelength from 500nm to that in methyl iodide?
@donerphysics
@donerphysics Жыл бұрын
That is what the 1.76 does.
@sachinprabhuram5765
@sachinprabhuram5765 3 жыл бұрын
I did a question on N19 that says that when there are two inversions like in 13:24 there is a path difference of 2d. How? Question is Q10 b) (ii) in N19 for reference
@yewweeyak3226
@yewweeyak3226 Жыл бұрын
Shouldn't you account for the refractive index of glass as well?
@donerphysics
@donerphysics Жыл бұрын
Neither of the interfering rays travel through the glass so no.
@vanessawertheim
@vanessawertheim 3 жыл бұрын
Thank you so much for this helpful video!
@donerphysics
@donerphysics 3 жыл бұрын
Glad it was helpful!
@ezu8501
@ezu8501 2 жыл бұрын
YOURE A GOD
@zhenghongchen8365
@zhenghongchen8365 2 жыл бұрын
Mr. Doner, in question 1( 21:00), which answer is the correct answer or all the answers are the correct answer? Please further clarify a little bit 😂 thanks
@donerphysics
@donerphysics 2 жыл бұрын
At the thicknesses indicated the reflected rays will be in phase.
@jasonfong7394
@jasonfong7394 Жыл бұрын
Hi, sorry to bother you. I was wondering if you could break down the optically dense --> rare --> dense scenario for thin film interference. Thanks!
@donerphysics
@donerphysics Жыл бұрын
most common is air-film-air where we get a single phase inversion. the other situation you will see is air- more dense-even more dense, where there are two phase inversions which is equivalent to none at all.
@ezu8501
@ezu8501 2 жыл бұрын
Hey, can you explain what the m represents? do you have any videos explaining?
@donerphysics
@donerphysics 2 жыл бұрын
It is explained thoroughly in previous videos.. We get constructive interference when the the path difference is an integer number of wavelengths. m represents that integer.
@andreshiguematu9207
@andreshiguematu9207 4 жыл бұрын
Souldn't you divide the index of refraction of the film by the index of refraction of the surroundings in order to appropriately convert lamba from the film to lamba in the surroundings?
@donerphysics
@donerphysics 4 жыл бұрын
Yes, we must generally use the relative index of refraction. The example was the common case in which the first medium is air for which n=1.
@sidninja9518
@sidninja9518 3 жыл бұрын
23:38 would the answer be 189nm because rearranging the equation gives us t=(1/2lambda)/2n? I’m confused how the denominator becomes 4n
@donerphysics
@donerphysics 3 жыл бұрын
There is a 2 on the LHS as well as the RHS. The one the LHS represents a half wavelength of path difference that is necessary for destructive interference. The one on the RHS represents twice the thickness since the reflected beams travel through the thickness twice.
@letseat1098
@letseat1098 2 жыл бұрын
Sir, what are the factors affecting the thin film and how to protect the thin film?
@donerphysics
@donerphysics 2 жыл бұрын
What thin film are you referring to?
@justalittlebitoflove6520
@justalittlebitoflove6520 3 жыл бұрын
In the first example, I considered the air as the first medium, and thus concluded two inversions. Why is air not considered in this case?
@donerphysics
@donerphysics 3 жыл бұрын
Good question. Most problems involve air. There are two ways to know. One would be by having experiences with this type of application (which is unlikely) , and the other would be to recognize that no thickness is given for the glass and therefore we can not treat it as a thin film, just as we do not treat the air as a thin film.
@aviratthakor8974
@aviratthakor8974 3 жыл бұрын
I am totally lost. Why at 10:35 do we need to multiply by the index of refraction of the film?
@aviratthakor8974
@aviratthakor8974 3 жыл бұрын
And why would there be constructive or destructive interference if there are 0 or 2 inversions (18:09)? Wouldn't the reflected rays be in the same phase?
@donerphysics
@donerphysics 3 жыл бұрын
The wavelength changes whenever the medium changes. The higher the index of refraction the shorter the wavelength. The path difference is created when the light travels through the film, so we must use the wavelength within the film.
@donerphysics
@donerphysics 3 жыл бұрын
@@aviratthakor8974 Good. O or 2 inversions has the same effect. To get the rays out of phase we need the light that travels through the film to travel an extra half wavelength or one and one half wavelengths etc.
@taliyaloz8878
@taliyaloz8878 7 жыл бұрын
Great content! Thank you!!
@engila9295
@engila9295 5 жыл бұрын
Hello! Thank you for the video but I have 2 questions: 1. What does the m stand for in the equation? 2. Why are the interference equations still applied for 0 or 2 inversions? I thought interference patterns only occured for 1 inversion.
@donerphysics
@donerphysics 5 жыл бұрын
1. m can be any number 0,1,2,3 etc., so m is a counter. Waves produce destructive interference when they are out of phase by half of a wavelength, or 1 and a half wavelengths etc. 2. No, we get stable interference patterns when two waves of the same frequency pass through each other.
@engila9295
@engila9295 5 жыл бұрын
Chris Doner but why was m=0 used when thin film’s thickness was assumed to be minimal? How would the phase difference and thickness denote the number for m?
@donerphysics
@donerphysics 5 жыл бұрын
At what time in the video are you referring to?
@engila9295
@engila9295 5 жыл бұрын
Chris Doner around 20:10
@donerphysics
@donerphysics 5 жыл бұрын
@@engila9295 20:10 is the summary. Is there a specific case that you are referring to? I have already answered your question in general.
@tauhid9983
@tauhid9983 5 жыл бұрын
@22.55 Hey Mr Doner, could you please explain why there are 2 inversions
@donerphysics
@donerphysics 5 жыл бұрын
Any time the index of refraction increases we get an inversion. The index increases twice.
@tauhid9983
@tauhid9983 5 жыл бұрын
@@donerphysics So if for example the index of refraction of the film is equal to the index of refraction of the medium through which light come out of the film, would we get one inversion?
@donerphysics
@donerphysics 5 жыл бұрын
@@tauhid9983 If they truly have the same index, they are the same material and there is no reflection.
@tauhid9983
@tauhid9983 5 жыл бұрын
@@donerphysics therefore no inversion... hence if there's reflection between mediums, like you've marked the 2 point of reflection on the video, there will be 2 inversions
@tauhid9983
@tauhid9983 5 жыл бұрын
Hell Mr Donner, could you explan whether or not, with respect to thin films, will be an inversion for an air wedge
@donerphysics
@donerphysics 5 жыл бұрын
The (somewhat) relevant reflections are glass to air (no inversion) and air to glass (an inverstion), however, for air wedges the inversion isn't important since you are usually just interested in the idea that everytime a new fringe is produced, the path difference (2t) has increased by one wavelength.
@tauhid9983
@tauhid9983 5 жыл бұрын
@@donerphysics So in IB question... we can take any equation 2t=nλ/2 or 2t=nλ..... to find for example the thickness of the film
Thin Film Interference - IB Physics
17:54
Mr. Brown
Рет қаралды 6 М.
Air Sigma Girl #sigma
0:32
Jin and Hattie
Рет қаралды 45 МЛН
#behindthescenes @CrissaJackson
0:11
Happy Kelli
Рет қаралды 27 МЛН
How to Solve Thin Film Interference Problems in Physics
13:48
Dan the Tutor
Рет қаралды 35 М.
IB Physics: Polarization
14:42
Chris Doner
Рет қаралды 45 М.
Thin film interference and the beauty of soap bubbles
5:25
Bryan Rolfe
Рет қаралды 86 М.
Simple Thin Film Thickness Measurement with Spectrometer
7:55
Sam Zeloof
Рет қаралды 31 М.
27 Inteference & Diffraction - Thin film interference
14:13
Cogverse Academy
Рет қаралды 83 М.
How wiggling charges give rise to light | Optics puzzles 2
21:33
3Blue1Brown
Рет қаралды 915 М.
How big is a visible photon?
20:34
Huygens Optics
Рет қаралды 735 М.
Interference Between Two Glass Plates
7:35
Physics Ninja
Рет қаралды 30 М.
Air Sigma Girl #sigma
0:32
Jin and Hattie
Рет қаралды 45 МЛН